Logarithms: Solving for Log(3) 25 and Finding Log(3) 75 and Log(5) 75

  • Thread starter Thread starter craig100
  • Start date Start date
  • Tags Tags
    Logarithms Table
Click For Summary
To solve for log(3) 25, it has been established that 2 < log(3) 25 < 3 by comparing powers of 3, confirming that 9 < 25 < 27. This indicates that log(3) 25 is between 2 and 3. Additionally, to find the ceiling of log(3) 75, one can use the relationship log(5) 75 = log(3) 75 / log(3) 5, applying the change of base formula. The discussion highlights the importance of understanding logarithmic properties without relying on calculators or tables. Overall, the thread emphasizes foundational logarithmic concepts and their applications.
craig100
Messages
7
Reaction score
0
Hello there, I am wondering if you could offer some help on this questoin, I have been attempting to work through it, however cannot see exactly what route I should be taking to try and solve it;
Without using a calculator or table, show that;
2 < log(3) 25 < 3 (log to the base 3 of 25)
and hence find the ceiling of log(3) 75
Also calculate log(5) 75 in terms of log(3)5
Thankyou for any help you can provide.
Craig :)

Edit:

Sorry i forgot to mention, what i have done so far is;

Shown that 3^3 = 27 and 3^2 = 9, this is simply setting log(3)x = 3 and 2 and i get a values in which 25 lies in

9 < 25 < 27
 
Last edited:
Physics news on Phys.org
My logarithm knowledge is kind of sketchy but I think this is right:

x = log_3 25

3^x = 25

If x were 3 then RHS would be 27, if it were 2 then RHS would be 9, since 25 is between these values then so is x.

Q2:

I think there is a rule that goes something like this (someone please confirm):

log_x y = \frac{log_a y}{log_a x} (where a can be anything you like).

So

log_5 75 = \frac{log_3 75}{log_3 5}
 
Last edited:
Question: A clock's minute hand has length 4 and its hour hand has length 3. What is the distance between the tips at the moment when it is increasing most rapidly?(Putnam Exam Question) Answer: Making assumption that both the hands moves at constant angular velocities, the answer is ## \sqrt{7} .## But don't you think this assumption is somewhat doubtful and wrong?

Similar threads

  • · Replies 1 ·
Replies
1
Views
1K
  • · Replies 10 ·
Replies
10
Views
3K
  • · Replies 22 ·
Replies
22
Views
4K
  • · Replies 3 ·
Replies
3
Views
2K
  • · Replies 1 ·
Replies
1
Views
1K
  • · Replies 9 ·
Replies
9
Views
3K
  • · Replies 15 ·
Replies
15
Views
3K
  • · Replies 4 ·
Replies
4
Views
2K
  • · Replies 4 ·
Replies
4
Views
2K
  • · Replies 3 ·
Replies
3
Views
3K